1
$\begingroup$

Assume that $J$ is the interval $(-\pi,\pi]$. For $k=1,\ldots,2n$, suppose that $\lambda_k$s are real functions on $J$ with $|\lambda_k|=1$, meaning that $\lambda_k(t)$ is either $-1$ or $1$ where $t\in J$.

Let us fix real numbers $b_1,\ldots, b_{2n}$. We define,

$$f:J\to \mathbb{C} : f(t)=\sum_{k=1}^{2n}b_{k}\lambda_k(t)e^{ikt}$$

Q. Is it valid to assert that the cardinality of the set of roots of the function $f$ does not exceed $2n$?

$\endgroup$
0

1 Answer 1

2
$\begingroup$

Let $z=e^{it}$ so taking out a $z$ factor we get an expression of the type $\sum_{k=0}^{2n-1}b_{k+1} \lambda_{k+1}(z)z^k, \lambda_k(z)=\pm 1$ and we need to find roots on the unit circle.

Take $n=3$ and $b_1=0, b_2=...b_6=1$ then simplifying another $z$ we get $\sum_{k=0}^{4} \lambda_{k+2}(z)z^k, \lambda_k(z)=\pm 1$ so if $\omega_{1,2,3,4}$ are the non real roots of $z^5=1$ and $\eta_{1,2,3,4}$ are the non real roots of $z^5=-1$ we can choose $\lambda_k(w_{1,2,3,4})=1$ and $\lambda_k(\eta_{1,2,3,4})=(-1)^k$ and then clearly all those $8$ numbers are unit circle roots of the given $f$ so the OP's question has a negative answer.

Similarly, we can get for any $n$ an example with $4n-4$ roots.

However, this begs the question if the equation above which has always finitely many roots bound by $(2n-1)2^{2n}$ (as the choices of $\pm 1$ are $2^{2n}$ for fixed $n, b_k$ and each gives a polynomial of degree at most $2n-1$ as noted), has always the number of roots bound by some polynomial function of $n$ and what is the minimum degree of such- eg if a linear bound like the one asked in the OP but slightly larger like say $4n$ holds

$\endgroup$

You must log in to answer this question.

Start asking to get answers

Find the answer to your question by asking.

Ask question

Explore related questions

See similar questions with these tags.